Discussion

Which one of the following, if true, would most support the television executives' argument?
(A)Most costs of production and distribution of products typically advertised on television are expected to rise 3 to 7 percent in the next year.
(B)...
(C)...
(D)...
(E)...
(F)...
*This question is included in Practice Set: "Stronger" Questions, Set 1, question #10

The solution is

Posted: 08/13/2012 12:36
What is the connection with the argument?
Image Not Available
Contributor
Posted: 09/09/2012 23:38
Hi, Zana -

For this problem, the goal is to find the answer that "best supports" the argument -- that is, the additional information that goes farthest in making the argument convincing.

In this case, the argument as given has a large hole in it: it does not give any facts to support its assertion that "in spite of this increase, advertisers will continue to profit from television advertising." In fact, if the increased cost does not correspond to any increase in revenue, the assertion may actually be false; I.e., it is possible that television advertising will cost more than it is worth.

Thus the strongest (and thus correct) answer will be one that addresses this concern. In other words, the correct answer will suggest some mechanism by which the advertisers' returns may be increased in the fall.

The only answer that accomplishes this is 'E': if each viewer watches more television, then the average number of viewers for each advertisement will increase, thus presumably making the advertisement more effective.

I hope this helps; if you have further questions, please don't hesitate to post again.

Best,
Lyn
Posted: 09/13/2012 14:05
thanks!
| Edit

You need to be signed in to perform that action.

Sign In